Search found 17 matches

Return to advanced search

by wayne_palmer10
Wed Sep 02, 2009 11:52 am
 
Forum: Section #2
Topic: Q10 - Cafeteria patron: The apples sold
Replies: 12
Views: 5364
Jump to post

Q10 - Cafeteria patron: The apples sold

This is a necessary assumption question. I selected (B) but don't undertand why (A) is correct. Is (B) wrong because it's too general (i.e. it doesn;t talk about the cafeteria's apples)?
by wayne_palmer10
Thu Oct 01, 2009 10:19 pm
 
Forum: Section #2
Topic: Q11 - A gift is not generous unless
Replies: 20
Views: 5834
Jump to post

PT39, S2, Q11 - A gift is not generous unless

This type of principle question throws me off. Given this type of question, I think that we can only conclude the necessary condition and the negation of the sufficient condition. So, in this specific question, we can conclude that that a gift is intended to benefit the recepient and is worth more.....
by wayne_palmer10
Thu Oct 01, 2009 10:07 pm
 
Forum: Section #2
Topic: Q23 - Politician: Nobody can deny that
Replies: 5
Views: 3836
Jump to post

Q23 - Politician: Nobody can deny that

I believe that this is a strengthen question. The sufficient and necessary terms in the answer choices threw me off a bit. I think that "only if" introduces a necessary condition, but I'm not sure why (C) is correct.
by wayne_palmer10
Fri Sep 25, 2009 5:50 pm
 
Forum: Section #4
Topic: Q26 - Smoking in bed has long
Replies: 10
Views: 6056
Jump to post

PT 20, S4, Q26 - Smoking in bed has long

I wasn't sure why (B) solved the discrepancy. Is it because it only talks about when?
by wayne_palmer10
Fri Sep 25, 2009 5:46 pm
 
Forum: Section #4
Topic: Q13 - Unlike other primroses, self-pollinating primroses
Replies: 8
Views: 2859
Jump to post

Q13 - Unlike other primroses, self-pollinating primroses

What would be the best way to solve a lengthy question like this one? I wasn't sure why (B) was correct.
by wayne_palmer10
Fri Sep 25, 2009 5:16 pm
 
Forum: Section #2
Topic: Q10 - The government of Penglai
Replies: 14
Views: 4867
Jump to post

Q10 - The government of Penglai

I didn't notice the assumption flaw in the stimulus, so I was surprised that (B) was the correct answer. Any advice? Thanks.
by wayne_palmer10
Fri Sep 25, 2009 5:14 pm
 
Forum: Section #2
Topic: Q16 - Electronic media are bound to
Replies: 13
Views: 5230
Jump to post

PT37, S2, Q16 - Electronic media are bound to

I didn't see the sufficient/necessary flaw in the stimulus, so I was surprised that (D) was the correct answer. Could you please explain this? Thanks.
by wayne_palmer10
Fri Sep 25, 2009 5:02 pm
 
Forum: Section #1
Topic: Q26 - The vast majority of a person's
Replies: 9
Views: 5577
Jump to post

PT27, S1, Q26 - The vast majority of a person's

I know that this is a parallel reasoning question, but I can't seem to figure out by (D) is right. Any help would be great. Thanks.
by wayne_palmer10
Fri Sep 25, 2009 4:59 pm
 
Forum: Section #4
Topic: Q9 - Manuscripts written by first-time
Replies: 9
Views: 4545
Jump to post

Q9 - Manuscripts written by first-time

I know that this is a parallel reasoning question, but I can't seem to figure out by (B) is right. Is it necessary to parallel the "except" in the stimulus with the "unless" in the correct answer choice? That might have thrown me off. Any help would be great. Thanks.
by wayne_palmer10
Tue Aug 25, 2009 5:28 pm
 
Forum: Section #1
Topic: Q18 - People cannot be morally
Replies: 19
Views: 8562
Jump to post

Q18 - People cannot be morally

I was wondering if you'd diagram this question? I was thinking (A) would be correct but the language is too strong. Not exactly sure why (E) is correct.
by wayne_palmer10
Wed Sep 02, 2009 11:49 am
 
Forum: Section #2
Topic: Q14 - In 1980, Country A had
Replies: 3
Views: 3596
Jump to post

Q14 - In 1980, Country A had

This is a necessary assumption question. Since the stimulus used concrete numbers, I was looking for an answer choice that did the same. I ultimately guessed with (D) but can't seem to figure out why it's correct.
by wayne_palmer10
Wed Sep 02, 2009 11:47 am
 
Forum: Section #3
Topic: Q14 - If the proposed tax reduction
Replies: 15
Views: 3997
Jump to post

Q14 - If the proposed tax reduction

This is a sufficient assumption question. I tried diagramming the stimulus in order to find the missing premise but had some difficulty.
by wayne_palmer10
Wed Sep 02, 2009 11:38 am
 
Forum: Section #1
Topic: Q19 - Anger in response to insults
Replies: 4
Views: 4021
Jump to post

Q19 - Anger in response to insults

This is a sufficient assumption question. The correct answer is (C), yet I did not ultimately choose this answer because of the word "any." I felt that this was too strong. Any explanation for this question would be appreciated!
by wayne_palmer10
Wed Sep 02, 2009 11:35 am
 
Forum: Section #2
Topic: Q12 - Photovoltaic power plants produce electricity
Replies: 4
Views: 3434
Jump to post

PT3, S1, Q12 Photovoltaic power plants produce electricity

Based on the question stem, this is a sufficient assumption question. I had trouble with the answer choices. All of them seemed so similar with the terminology and with the way that they were presented. How would you go about solving this problem? Thanks!
by wayne_palmer10
Wed Sep 02, 2009 11:31 am
 
Forum: Section #1
Topic: Q22 - At the end of the year,
Replies: 7
Views: 4102
Jump to post

Q22 - At the end of the year,

I realize that this is a sufficient assumption question. I just wasn't sure of the best way of attacking this question. Numbers and percentages seem to trip me up.
by wayne_palmer10
Tue Aug 25, 2009 5:38 pm
 
Forum: Section #4
Topic: Q19 - Nearly all mail that is
Replies: 19
Views: 12621
Jump to post

Q19 - Nearly all mail that is

What would be the best way to attack this problem? Would you diagram it? I tried to, but the "nearly all" threw me off a bit.
by wayne_palmer10
Tue Aug 25, 2009 5:33 pm
 
Forum: PrepTest 44 - October 2004 LSAT Answers & Explanations - LR
Topic: PT44, S1, Q14 The economy is doing badly
Replies: 2
Views: 890
Jump to post

PT44, S1, Q14 The economy is doing badly

How would you go about diagramming this one? "Their occurrence together..." seems to indicate a sufficient condition, but the previous sentence threw me off a bit.